Aufgaben:Aufgabe 3.2Z: Besselspektrum: Unterschied zwischen den Versionen

Aus LNTwww
Wechseln zu:Navigation, Suche
K (Textersetzung - „*Sollte die Eingabe des Zahlenwertes „0” erforderlich sein, so geben Sie bitte „0.” ein.“ durch „ “)
 
(6 dazwischenliegende Versionen von einem anderen Benutzer werden nicht angezeigt)
Zeile 8: Zeile 8:
 
Beispielsweise kann man das äquivalente Tiefpass–Signal am Ausgang eines Winkelmodulators (PM, FM) in dieser Form darstellen, wenn man geeignete Normierungen vornimmt.
 
Beispielsweise kann man das äquivalente Tiefpass–Signal am Ausgang eines Winkelmodulators (PM, FM) in dieser Form darstellen, wenn man geeignete Normierungen vornimmt.
  
Die Fourierreihendarstellung lautet mit $T_0 = 2π/ω_0$:
+
*Die Fourierreihendarstellung lautet mit  $T_0 = 2π/ω_0$:
 
:$$x(t)  =  \sum_{n = - \infty}^{+\infty}D_n \cdot{\rm e}^{\hspace{0.05cm}{\rm j} \hspace{0.05cm}\cdot \hspace{0.05cm}n \hspace{0.05cm}\cdot \hspace{0.05cm}\omega_{\rm 0} \hspace{0.05cm}\cdot \hspace{0.05cm} t }\hspace{0.05cm},$$  
 
:$$x(t)  =  \sum_{n = - \infty}^{+\infty}D_n \cdot{\rm e}^{\hspace{0.05cm}{\rm j} \hspace{0.05cm}\cdot \hspace{0.05cm}n \hspace{0.05cm}\cdot \hspace{0.05cm}\omega_{\rm 0} \hspace{0.05cm}\cdot \hspace{0.05cm} t }\hspace{0.05cm},$$  
 
:$$ D_n  =  \frac{1}{T_0}\cdot \int_{- T_0/2}^{+T_0/2}x(t) \cdot{\rm e}^{\hspace{0.05cm}{-\rm j} \hspace{0.05cm}\cdot \hspace{0.05cm}n \hspace{0.05cm}\cdot \hspace{0.05cm}\omega_{\rm 0} \hspace{0.05cm} \cdot \hspace{0.05cm} t }\hspace{0.1cm}{\rm d}t \hspace{0.05cm}.$$
 
:$$ D_n  =  \frac{1}{T_0}\cdot \int_{- T_0/2}^{+T_0/2}x(t) \cdot{\rm e}^{\hspace{0.05cm}{-\rm j} \hspace{0.05cm}\cdot \hspace{0.05cm}n \hspace{0.05cm}\cdot \hspace{0.05cm}\omega_{\rm 0} \hspace{0.05cm} \cdot \hspace{0.05cm} t }\hspace{0.1cm}{\rm d}t \hspace{0.05cm}.$$
Diese komplexen Fourierkoeffizienten können mit Hilfe der Besselfunktionen erster Art und n–ter Ordnung ausgedrückt werden:
+
*Diese komplexen Fourierkoeffizienten können mit Hilfe der Besselfunktionen erster Art und  $n$–ter Ordnung ausgedrückt werden:
 
:$${\rm J}_n (\eta) = \frac{1}{2\pi}\cdot \int_{-\pi}^{+\pi} {{\rm e}^{\hspace{0.05cm}{\rm j}\hspace{0.05cm}\cdot \hspace{0.05cm}(\eta \hspace{0.05cm}\cdot \hspace{0.05cm}\sin(\alpha) -\hspace{0.05cm} n \hspace{0.05cm}\cdot \hspace{0.05cm}\alpha)}}\hspace{0.1cm}{\rm d}\alpha \hspace{0.05cm}.$$
 
:$${\rm J}_n (\eta) = \frac{1}{2\pi}\cdot \int_{-\pi}^{+\pi} {{\rm e}^{\hspace{0.05cm}{\rm j}\hspace{0.05cm}\cdot \hspace{0.05cm}(\eta \hspace{0.05cm}\cdot \hspace{0.05cm}\sin(\alpha) -\hspace{0.05cm} n \hspace{0.05cm}\cdot \hspace{0.05cm}\alpha)}}\hspace{0.1cm}{\rm d}\alpha \hspace{0.05cm}.$$
Diese sind in der Grafik im Bereich $0 ≤ η ≤ 5$ dargestellt. Für negative Werte von $n$ erhält man:
+
*Diese sind in der Grafik im Bereich  $0 ≤ η ≤ 5$  dargestellt.  Für negative Werte von  $n$  erhält man:
 
:$${\rm J}_{-n} (\eta) = (-1)^n \cdot {\rm J}_{n} (\eta)\hspace{0.05cm}.$$
 
:$${\rm J}_{-n} (\eta) = (-1)^n \cdot {\rm J}_{n} (\eta)\hspace{0.05cm}.$$
Die Reihendarstellung der Besselfunktionen lautet:
+
*Die Reihendarstellung der Besselfunktionen lautet:
 
:$${\rm J}_n (\eta) = \sum\limits_{k=0}^{\infty}\frac{(-1)^k \cdot (\eta/2)^{n \hspace{0.05cm} + \hspace{0.05cm} 2 \hspace{0.02cm}\cdot \hspace{0.05cm}k}}{k! \cdot (n+k)!} \hspace{0.05cm}.$$
 
:$${\rm J}_n (\eta) = \sum\limits_{k=0}^{\infty}\frac{(-1)^k \cdot (\eta/2)^{n \hspace{0.05cm} + \hspace{0.05cm} 2 \hspace{0.02cm}\cdot \hspace{0.05cm}k}}{k! \cdot (n+k)!} \hspace{0.05cm}.$$
Sind die Funktionswerte für $n = 0$ und $n = 1$ bekannt, so können daraus die Besselfunktionen für $n ≥ 2$ iterativ ermittelt werden:
+
*Sind die Funktionswerte für  $n = 0$  und  $n = 1$  bekannt, so können daraus die Besselfunktionen für  $n ≥ 2$  iterativ ermittelt werden:
 
:$${\rm J}_n (\eta) = \frac{2 \cdot (n-1)}{\eta} \cdot {\rm J}_{n-1} (\eta) - {\rm J}_{n-2} (\eta) \hspace{0.05cm}.$$
 
:$${\rm J}_n (\eta) = \frac{2 \cdot (n-1)}{\eta} \cdot {\rm J}_{n-1} (\eta) - {\rm J}_{n-2} (\eta) \hspace{0.05cm}.$$
 +
 +
 +
 +
 +
 +
  
  
  
 
''Hinweise:''  
 
''Hinweise:''  
*Die Aufgabe gehört zum  Kapitel [[Modulationsverfahren/Phasenmodulation_(PM)|Phasenmodulation]].
+
*Die Aufgabe gehört zum  Kapitel  [[Modulationsverfahren/Phasenmodulation_(PM)|Phasenmodulation]].
*Bezug genommen wird insbesondere auf die Seite  [[Modulationsverfahren/Phasenmodulation_(PM)#.C3.84quivalentes_TP.E2.80.93Signal_bei_Phasenmodulation|Äquivalentes Tiefpass-Signal bei Phasenmodulation]].
+
*Bezug genommen wird insbesondere auf die Seite  [[Modulationsverfahren/Phasenmodulation_(PM)#.C3.84quivalentes_TP.E2.80.93Signal_bei_Phasenmodulation|Äquivalentes Tiefpass-Signal bei Phasenmodulation]].
 
*Die Werte der Besselfunktionen findet man in Formelsammlungen  in tabellarischer Form.  
 
*Die Werte der Besselfunktionen findet man in Formelsammlungen  in tabellarischer Form.  
*Sie können zur Lösung dieser Aufgabe auch das Interaktionsmodul [[Werte der Besselfunktion erster Art und n–ter Ordnung]] nutzen.
+
*Sie können zur Lösung dieser Aufgabe auch das interaktive Applet  [[Applets:Besselfunktionen_erster_Art| Besselfunktion erster Art]]  benutzen.
 
   
 
   
  
Zeile 34: Zeile 40:
 
<quiz display=simple>
 
<quiz display=simple>
  
{Welche Eigenschaften besitzt das Signal $x(t)$?
+
{Welche Eigenschaften besitzt das Signal &nbsp;$x(t)$?
 
|type="[]"}
 
|type="[]"}
- $x(t)$ ist für alle Zeiten $t$ imaginär.
+
- $x(t)$&nbsp; ist für alle Zeiten &nbsp;$t$&nbsp; imaginär.
+ $x(t)$ ist periodisch.
+
+ $x(t)$&nbsp; ist periodisch.
- Die Spektralfunktion $X(f)$ erhält man über das Fourierintegral.
+
- Die Spektralfunktion &nbsp;$X(f)$&nbsp; erhält man über das Fourierintegral.
  
{Schreiben Sie die Fourierkoeffizienten $D_n$ mit den Besselfunktionen erster Art &nbsp; &rArr; &nbsp; ${\rm J}_n(η)$.  
+
{Schreiben Sie die Fourierkoeffizienten &nbsp;$D_n$&nbsp; mit den Besselfunktionen erster Art &nbsp; &rArr; &nbsp; ${\rm J}_n(η)$.&nbsp; Welche Zusammenhänge sind zu erkennen?
<br>Welche Zusammenhänge sind zu erkennen?
 
 
|type="[]"}
 
|type="[]"}
- Alle $D_n$ sind gleich ${\rm J}_η(0)$.
+
- Alle &nbsp;$D_n$&nbsp; sind gleich &nbsp;${\rm J}_η(0)$.
+ Es gilt $D_n = {\rm J}_n(η)$.
+
+ Es gilt &nbsp;$D_n = {\rm J}_n(η)$.
- Es gilt $D_n = -{\rm J}_η(n)$.
+
- Es gilt &nbsp;$D_n = -{\rm J}_η(n)$.
  
 
{ Welche Eigenschaften besitzen die Fourierkoeffizienten?
 
{ Welche Eigenschaften besitzen die Fourierkoeffizienten?
|type="[]"}
+
|type="()"}
+  Alle $D_n$ sind rein reell.
+
+  Alle&nbsp; $D_n$&nbsp; sind rein reell.
-  Alle $D_n$ sind rein imaginär.
+
-  Alle&nbsp; $D_n$&nbsp; sind rein imaginär.
  
{Für $η = 2$ lauten die Koeffizienten $D_0 = 0.224$ und $D_1 = 0.577$. Berechnen Sie hieraus die Koeffizienten $D_2$ und $D_3$.
+
{Für &nbsp;$η = 2$&nbsp; lauten die Koeffizienten &nbsp;$D_0 = 0.224$&nbsp; und &nbsp;$D_1 = 0.577$.&nbsp; Berechnen Sie hieraus die Koeffizienten &nbsp;$D_2$&nbsp; und &nbsp;$D_3$.
 
|type="{}"}
 
|type="{}"}
 
$D_2 \ = \ $ { 0.353 3% }  
 
$D_2 \ = \ $ { 0.353 3% }  
 
$D_3 \ = \ $ { 0.129 3% }  
 
$D_3 \ = \ $ { 0.129 3% }  
  
{Wie lauten die Fourierkoeffizienten $D_{-2}$ und $D_{-3}$ ?
+
{Wie lauten die Fourierkoeffizienten &nbsp;$D_{-2}$&nbsp; und &nbsp;$D_{-3}$ ?
 
|type="{}"}
 
|type="{}"}
 
$D_{-2} \ = \ $ { 0.353 3% }  
 
$D_{-2} \ = \ $ { 0.353 3% }  
Zeile 68: Zeile 73:
 
{{ML-Kopf}}
 
{{ML-Kopf}}
 
'''(1)'''&nbsp; Richtig ist nur der <u>zweite Lösungsvorschlag</u>:
 
'''(1)'''&nbsp; Richtig ist nur der <u>zweite Lösungsvorschlag</u>:
*$x(t)$ ist ein komplexes Signal, das nur in Ausnahmefällen reell wird, zum Beispiel zur Zeit $t = 0$. Ein rein imaginärer Wert (zu gewissen Zeiten) kann sich nur dann ergeben, wenn $η ≥ π/2$ ist &rArr; &nbsp;  Antwort 1 ist falsch.
+
*$x(t)$&nbsp; ist ein komplexes Signal, das nur in Ausnahmefällen reell wird, zum Beispiel zur Zeit&nbsp; $t = 0$.  
*Mit $T_0 = 2π/ω_0$ gilt beispielsweise:
+
*Ein rein imaginärer Wert (zu gewissen Zeiten) kann sich nur dann ergeben, wenn&nbsp; $η ≥ π/2$&nbsp; ist &nbsp; &rArr; &nbsp;  Antwort 1 ist falsch.
 +
*Mit&nbsp; $T_0 = 2π/ω_0$&nbsp; gilt beispielsweise:
 
:$$ x(t + k \cdot T_0)  =  {\rm e}^{\hspace{0.05cm}{\rm j} \hspace{0.05cm}\cdot \hspace{0.05cm}\eta \hspace{0.05cm}\cdot \hspace{0.05cm}\sin (\omega_{\rm 0} \hspace{0.05cm}\cdot \hspace{0.05cm} (t \hspace{0.05cm}+ \hspace{0.05cm} k \hspace{0.05cm}\cdot \hspace{0.05cm}T_0)) } =
 
:$$ x(t + k \cdot T_0)  =  {\rm e}^{\hspace{0.05cm}{\rm j} \hspace{0.05cm}\cdot \hspace{0.05cm}\eta \hspace{0.05cm}\cdot \hspace{0.05cm}\sin (\omega_{\rm 0} \hspace{0.05cm}\cdot \hspace{0.05cm} (t \hspace{0.05cm}+ \hspace{0.05cm} k \hspace{0.05cm}\cdot \hspace{0.05cm}T_0)) } =
 
   {\rm e}^{\hspace{0.05cm}{\rm j} \hspace{0.05cm}\cdot \hspace{0.05cm}\eta \hspace{0.05cm}\cdot \hspace{0.05cm}\sin (\omega_{\rm 0} \hspace{0.05cm}\cdot \hspace{0.05cm} t \hspace{0.05cm} + \hspace{0.05cm} k \hspace{0.05cm}\cdot \hspace{0.05cm} 2 \pi) } ={\rm e}^{\hspace{0.05cm}{\rm j} \hspace{0.05cm}\cdot \hspace{0.05cm}\eta \hspace{0.05cm}\cdot \hspace{0.05cm}\sin (\omega_{\rm 0} \hspace{0.05cm}\cdot \hspace{0.05cm} t \hspace{0.05cm} ) } = x(t)\hspace{0.05cm}.$$
 
   {\rm e}^{\hspace{0.05cm}{\rm j} \hspace{0.05cm}\cdot \hspace{0.05cm}\eta \hspace{0.05cm}\cdot \hspace{0.05cm}\sin (\omega_{\rm 0} \hspace{0.05cm}\cdot \hspace{0.05cm} t \hspace{0.05cm} + \hspace{0.05cm} k \hspace{0.05cm}\cdot \hspace{0.05cm} 2 \pi) } ={\rm e}^{\hspace{0.05cm}{\rm j} \hspace{0.05cm}\cdot \hspace{0.05cm}\eta \hspace{0.05cm}\cdot \hspace{0.05cm}\sin (\omega_{\rm 0} \hspace{0.05cm}\cdot \hspace{0.05cm} t \hspace{0.05cm} ) } = x(t)\hspace{0.05cm}.$$
*Dieses Signal ist periodisch. Zur Berechnung der Spektralfunktion muss die Fourierreihe und nicht das Fourierintegral herangezogen werden.  
+
*Dieses Signal ist periodisch.&nbsp; Zur Berechnung der Spektralfunktion muss die Fourierreihe und nicht das Fourierintegral herangezogen werden.  
 +
 
  
  
 
'''(2)'''&nbsp; Die Fourierkoeffizienten lauten:
 
'''(2)'''&nbsp; Die Fourierkoeffizienten lauten:
 
:$$ D_n = \frac{1}{T_0}\cdot \int_{- T_0/2}^{+T_0/2}{\rm e}^{\hspace{0.05cm}{\rm j} \hspace{0.05cm}\cdot \hspace{0.05cm}\eta \hspace{0.05cm}\cdot \hspace{0.05cm}\sin (\omega_{\rm 0} \hspace{0.05cm} \cdot \hspace{0.05cm} t) }\cdot{\rm e}^{\hspace{0.05cm}{-\rm j} \hspace{0.05cm}\cdot \hspace{0.05cm}n \hspace{0.05cm}\cdot \hspace{0.05cm}\omega_{\rm 0} \hspace{0.05cm} \cdot \hspace{0.05cm} t }\hspace{0.1cm}{\rm d}t \hspace{0.05cm}.$$
 
:$$ D_n = \frac{1}{T_0}\cdot \int_{- T_0/2}^{+T_0/2}{\rm e}^{\hspace{0.05cm}{\rm j} \hspace{0.05cm}\cdot \hspace{0.05cm}\eta \hspace{0.05cm}\cdot \hspace{0.05cm}\sin (\omega_{\rm 0} \hspace{0.05cm} \cdot \hspace{0.05cm} t) }\cdot{\rm e}^{\hspace{0.05cm}{-\rm j} \hspace{0.05cm}\cdot \hspace{0.05cm}n \hspace{0.05cm}\cdot \hspace{0.05cm}\omega_{\rm 0} \hspace{0.05cm} \cdot \hspace{0.05cm} t }\hspace{0.1cm}{\rm d}t \hspace{0.05cm}.$$
Durch Zusammenfassen der beiden Terme und nach der Substitution $α = ω_0 · t$ erhält man:
+
*Durch Zusammenfassen der beiden Terme und nach der Substitution&nbsp; $α = ω_0 · t$&nbsp; erhält man:
 
:$$D_n = \frac{1}{2\pi}\cdot \int_{-\pi}^{+\pi} {{\rm e}^{\hspace{0.05cm}{\rm j}\hspace{0.05cm}\cdot \hspace{0.05cm}(\eta \hspace{0.05cm}\cdot \hspace{0.05cm}\sin(\alpha) -\hspace{0.05cm} n \hspace{0.05cm}\cdot \hspace{0.05cm}\alpha)}}\hspace{0.1cm}{\rm d}\alpha \hspace{0.05cm} = {\rm J}_n (\eta) .$$
 
:$$D_n = \frac{1}{2\pi}\cdot \int_{-\pi}^{+\pi} {{\rm e}^{\hspace{0.05cm}{\rm j}\hspace{0.05cm}\cdot \hspace{0.05cm}(\eta \hspace{0.05cm}\cdot \hspace{0.05cm}\sin(\alpha) -\hspace{0.05cm} n \hspace{0.05cm}\cdot \hspace{0.05cm}\alpha)}}\hspace{0.1cm}{\rm d}\alpha \hspace{0.05cm} = {\rm J}_n (\eta) .$$
Richtig ist also der <u>zweite Lösungsvorschlag</u>.
+
*Richtig ist also der <u>zweite Lösungsvorschlag</u>.
 +
 
  
  
Zeile 85: Zeile 93:
 
:$$D_n  =  \frac{1}{2\pi}\cdot \int_{-\pi}^{+\pi} {\cos( \eta \cdot \sin(\alpha) - n \cdot \alpha)}\hspace{0.1cm}{\rm d}\alpha +  
 
:$$D_n  =  \frac{1}{2\pi}\cdot \int_{-\pi}^{+\pi} {\cos( \eta \cdot \sin(\alpha) - n \cdot \alpha)}\hspace{0.1cm}{\rm d}\alpha +  
 
\frac{\rm  j}{2\pi}\cdot \int_{-\pi}^{+\pi} {\sin( \eta \cdot \sin(\alpha) - n \cdot \alpha)}\hspace{0.1cm}{\rm d}\alpha \hspace{0.05cm}.$$
 
\frac{\rm  j}{2\pi}\cdot \int_{-\pi}^{+\pi} {\sin( \eta \cdot \sin(\alpha) - n \cdot \alpha)}\hspace{0.1cm}{\rm d}\alpha \hspace{0.05cm}.$$
Der Integrand des ersten Integrals ist eine gerade Funktion von α:
+
*Der Integrand des ersten Integrals ist eine gerade Funktion von&nbsp; $\alpha$:
 
:$$I_1 (-\alpha)  =  {\cos( \eta \cdot \sin(-\alpha) + n \cdot \alpha)} = {\cos( -\eta \cdot \sin(\alpha) + n \cdot \alpha)}=
 
:$$I_1 (-\alpha)  =  {\cos( \eta \cdot \sin(-\alpha) + n \cdot \alpha)} = {\cos( -\eta \cdot \sin(\alpha) + n \cdot \alpha)}=
 
   {\cos( \eta \cdot \sin(\alpha) - n \cdot \alpha)} = I_1 (\alpha) \hspace{0.05cm}.$$
 
   {\cos( \eta \cdot \sin(\alpha) - n \cdot \alpha)} = I_1 (\alpha) \hspace{0.05cm}.$$
Dagegen ist der zweite Integrand eine ungerade Funktion:
+
*Dagegen ist der zweite Integrand eine ungerade Funktion:
 
:$$I_2 (-\alpha)  =  {\sin( \eta \cdot \sin(-\alpha) + n \cdot \alpha)} = {\sin( -\eta \cdot \sin(\alpha) + n \cdot \alpha)}=
 
:$$I_2 (-\alpha)  =  {\sin( \eta \cdot \sin(-\alpha) + n \cdot \alpha)} = {\sin( -\eta \cdot \sin(\alpha) + n \cdot \alpha)}=
 
   -{\sin( \eta \cdot \sin(\alpha) - n \cdot \alpha)} = -I_2 (\alpha) \hspace{0.05cm}.$$
 
   -{\sin( \eta \cdot \sin(\alpha) - n \cdot \alpha)} = -I_2 (\alpha) \hspace{0.05cm}.$$
Somit verschwindet das zweite Integral und man erhält unter Berücksichtigung der Symmetrie:
+
*Somit verschwindet das zweite Integral und man erhält unter Berücksichtigung der Symmetrie:
 
:$$D_n = \frac{1}{\pi}\cdot \int_{0}^{\pi} {\cos( \eta \cdot \sin(\alpha) - n \cdot \alpha)}\hspace{0.1cm}{\rm d}\alpha \hspace{0.05cm}.$$
 
:$$D_n = \frac{1}{\pi}\cdot \int_{0}^{\pi} {\cos( \eta \cdot \sin(\alpha) - n \cdot \alpha)}\hspace{0.1cm}{\rm d}\alpha \hspace{0.05cm}.$$
Richtig ist somit der <u>Lösungsvorschlag 1</u>.
+
*Richtig ist somit der <u>Lösungsvorschlag 1</u>.
 +
 
  
  
'''(4)'''&nbsp; Entsprechend der iterativen Berechnungsformel gilt für $η = 2$:
+
'''(4)'''&nbsp; Entsprechend der iterativen Berechnungsformel gilt für&nbsp; $η = 2$:
 
:$$ D_2  =  D_1 - D_0 = 0.577 - 0.224 \hspace{0.15cm}\underline {= 0.353} \hspace{0.05cm},$$   
 
:$$ D_2  =  D_1 - D_0 = 0.577 - 0.224 \hspace{0.15cm}\underline {= 0.353} \hspace{0.05cm},$$   
 
:$$D_3  =  2 \cdot D_2 - D_1 = 2 \cdot 0.353 - 0.577 \hspace{0.15cm}\underline {= 0.129} \hspace{0.05cm}.$$
 
:$$D_3  =  2 \cdot D_2 - D_1 = 2 \cdot 0.353 - 0.577 \hspace{0.15cm}\underline {= 0.129} \hspace{0.05cm}.$$
Zeile 102: Zeile 111:
  
 
'''(5)'''&nbsp; Aufgrund der angegebenen Symmetriebeziehung gilt weiter:
 
'''(5)'''&nbsp; Aufgrund der angegebenen Symmetriebeziehung gilt weiter:
:$$ D{–2} = D_2\hspace{0.15cm}\underline {= 0.353} \hspace{0.05cm},$$   
+
:$$ D_{–2} = D_2\hspace{0.15cm}\underline {= 0.353} \hspace{0.05cm},$$   
:$$D{–3} = D_3  \hspace{0.15cm}\underline {= -0.129} \hspace{0.05cm}.$$
+
:$$D_{–3} = -D_3  \hspace{0.15cm}\underline {= -0.129} \hspace{0.05cm}.$$
  
  

Aktuelle Version vom 10. Juni 2020, 10:03 Uhr

Verlauf der Besselfunktionen

Wir betrachten das komplexe Signal

$$x(t) = {\rm e}^{\hspace{0.05cm}{\rm j} \hspace{0.05cm}\cdot \hspace{0.05cm}\eta \hspace{0.05cm}\cdot \hspace{0.05cm}\sin (\omega_{\rm 0} \hspace{0.05cm}\cdot \hspace{0.05cm} t) }\hspace{0.05cm}.$$

Beispielsweise kann man das äquivalente Tiefpass–Signal am Ausgang eines Winkelmodulators (PM, FM) in dieser Form darstellen, wenn man geeignete Normierungen vornimmt.

  • Die Fourierreihendarstellung lautet mit  $T_0 = 2π/ω_0$:
$$x(t) = \sum_{n = - \infty}^{+\infty}D_n \cdot{\rm e}^{\hspace{0.05cm}{\rm j} \hspace{0.05cm}\cdot \hspace{0.05cm}n \hspace{0.05cm}\cdot \hspace{0.05cm}\omega_{\rm 0} \hspace{0.05cm}\cdot \hspace{0.05cm} t }\hspace{0.05cm},$$
$$ D_n = \frac{1}{T_0}\cdot \int_{- T_0/2}^{+T_0/2}x(t) \cdot{\rm e}^{\hspace{0.05cm}{-\rm j} \hspace{0.05cm}\cdot \hspace{0.05cm}n \hspace{0.05cm}\cdot \hspace{0.05cm}\omega_{\rm 0} \hspace{0.05cm} \cdot \hspace{0.05cm} t }\hspace{0.1cm}{\rm d}t \hspace{0.05cm}.$$
  • Diese komplexen Fourierkoeffizienten können mit Hilfe der Besselfunktionen erster Art und  $n$–ter Ordnung ausgedrückt werden:
$${\rm J}_n (\eta) = \frac{1}{2\pi}\cdot \int_{-\pi}^{+\pi} {{\rm e}^{\hspace{0.05cm}{\rm j}\hspace{0.05cm}\cdot \hspace{0.05cm}(\eta \hspace{0.05cm}\cdot \hspace{0.05cm}\sin(\alpha) -\hspace{0.05cm} n \hspace{0.05cm}\cdot \hspace{0.05cm}\alpha)}}\hspace{0.1cm}{\rm d}\alpha \hspace{0.05cm}.$$
  • Diese sind in der Grafik im Bereich  $0 ≤ η ≤ 5$  dargestellt.  Für negative Werte von  $n$  erhält man:
$${\rm J}_{-n} (\eta) = (-1)^n \cdot {\rm J}_{n} (\eta)\hspace{0.05cm}.$$
  • Die Reihendarstellung der Besselfunktionen lautet:
$${\rm J}_n (\eta) = \sum\limits_{k=0}^{\infty}\frac{(-1)^k \cdot (\eta/2)^{n \hspace{0.05cm} + \hspace{0.05cm} 2 \hspace{0.02cm}\cdot \hspace{0.05cm}k}}{k! \cdot (n+k)!} \hspace{0.05cm}.$$
  • Sind die Funktionswerte für  $n = 0$  und  $n = 1$  bekannt, so können daraus die Besselfunktionen für  $n ≥ 2$  iterativ ermittelt werden:
$${\rm J}_n (\eta) = \frac{2 \cdot (n-1)}{\eta} \cdot {\rm J}_{n-1} (\eta) - {\rm J}_{n-2} (\eta) \hspace{0.05cm}.$$





Hinweise:


Fragebogen

1

Welche Eigenschaften besitzt das Signal  $x(t)$?

$x(t)$  ist für alle Zeiten  $t$  imaginär.
$x(t)$  ist periodisch.
Die Spektralfunktion  $X(f)$  erhält man über das Fourierintegral.

2

Schreiben Sie die Fourierkoeffizienten  $D_n$  mit den Besselfunktionen erster Art   ⇒   ${\rm J}_n(η)$.  Welche Zusammenhänge sind zu erkennen?

Alle  $D_n$  sind gleich  ${\rm J}_η(0)$.
Es gilt  $D_n = {\rm J}_n(η)$.
Es gilt  $D_n = -{\rm J}_η(n)$.

3

Welche Eigenschaften besitzen die Fourierkoeffizienten?

Alle  $D_n$  sind rein reell.
Alle  $D_n$  sind rein imaginär.

4

Für  $η = 2$  lauten die Koeffizienten  $D_0 = 0.224$  und  $D_1 = 0.577$.  Berechnen Sie hieraus die Koeffizienten  $D_2$  und  $D_3$.

$D_2 \ = \ $

$D_3 \ = \ $

5

Wie lauten die Fourierkoeffizienten  $D_{-2}$  und  $D_{-3}$ ?

$D_{-2} \ = \ $

$D_{-3} \ = \ $


Musterlösung

(1)  Richtig ist nur der zweite Lösungsvorschlag:

  • $x(t)$  ist ein komplexes Signal, das nur in Ausnahmefällen reell wird, zum Beispiel zur Zeit  $t = 0$.
  • Ein rein imaginärer Wert (zu gewissen Zeiten) kann sich nur dann ergeben, wenn  $η ≥ π/2$  ist   ⇒   Antwort 1 ist falsch.
  • Mit  $T_0 = 2π/ω_0$  gilt beispielsweise:
$$ x(t + k \cdot T_0) = {\rm e}^{\hspace{0.05cm}{\rm j} \hspace{0.05cm}\cdot \hspace{0.05cm}\eta \hspace{0.05cm}\cdot \hspace{0.05cm}\sin (\omega_{\rm 0} \hspace{0.05cm}\cdot \hspace{0.05cm} (t \hspace{0.05cm}+ \hspace{0.05cm} k \hspace{0.05cm}\cdot \hspace{0.05cm}T_0)) } = {\rm e}^{\hspace{0.05cm}{\rm j} \hspace{0.05cm}\cdot \hspace{0.05cm}\eta \hspace{0.05cm}\cdot \hspace{0.05cm}\sin (\omega_{\rm 0} \hspace{0.05cm}\cdot \hspace{0.05cm} t \hspace{0.05cm} + \hspace{0.05cm} k \hspace{0.05cm}\cdot \hspace{0.05cm} 2 \pi) } ={\rm e}^{\hspace{0.05cm}{\rm j} \hspace{0.05cm}\cdot \hspace{0.05cm}\eta \hspace{0.05cm}\cdot \hspace{0.05cm}\sin (\omega_{\rm 0} \hspace{0.05cm}\cdot \hspace{0.05cm} t \hspace{0.05cm} ) } = x(t)\hspace{0.05cm}.$$
  • Dieses Signal ist periodisch.  Zur Berechnung der Spektralfunktion muss die Fourierreihe und nicht das Fourierintegral herangezogen werden.


(2)  Die Fourierkoeffizienten lauten:

$$ D_n = \frac{1}{T_0}\cdot \int_{- T_0/2}^{+T_0/2}{\rm e}^{\hspace{0.05cm}{\rm j} \hspace{0.05cm}\cdot \hspace{0.05cm}\eta \hspace{0.05cm}\cdot \hspace{0.05cm}\sin (\omega_{\rm 0} \hspace{0.05cm} \cdot \hspace{0.05cm} t) }\cdot{\rm e}^{\hspace{0.05cm}{-\rm j} \hspace{0.05cm}\cdot \hspace{0.05cm}n \hspace{0.05cm}\cdot \hspace{0.05cm}\omega_{\rm 0} \hspace{0.05cm} \cdot \hspace{0.05cm} t }\hspace{0.1cm}{\rm d}t \hspace{0.05cm}.$$
  • Durch Zusammenfassen der beiden Terme und nach der Substitution  $α = ω_0 · t$  erhält man:
$$D_n = \frac{1}{2\pi}\cdot \int_{-\pi}^{+\pi} {{\rm e}^{\hspace{0.05cm}{\rm j}\hspace{0.05cm}\cdot \hspace{0.05cm}(\eta \hspace{0.05cm}\cdot \hspace{0.05cm}\sin(\alpha) -\hspace{0.05cm} n \hspace{0.05cm}\cdot \hspace{0.05cm}\alpha)}}\hspace{0.1cm}{\rm d}\alpha \hspace{0.05cm} = {\rm J}_n (\eta) .$$
  • Richtig ist also der zweite Lösungsvorschlag.


(3)  Mit dem Satz von Euler können die Fourierkoeffizienten wie folgt dargestellt werden:

$$D_n = \frac{1}{2\pi}\cdot \int_{-\pi}^{+\pi} {\cos( \eta \cdot \sin(\alpha) - n \cdot \alpha)}\hspace{0.1cm}{\rm d}\alpha + \frac{\rm j}{2\pi}\cdot \int_{-\pi}^{+\pi} {\sin( \eta \cdot \sin(\alpha) - n \cdot \alpha)}\hspace{0.1cm}{\rm d}\alpha \hspace{0.05cm}.$$
  • Der Integrand des ersten Integrals ist eine gerade Funktion von  $\alpha$:
$$I_1 (-\alpha) = {\cos( \eta \cdot \sin(-\alpha) + n \cdot \alpha)} = {\cos( -\eta \cdot \sin(\alpha) + n \cdot \alpha)}= {\cos( \eta \cdot \sin(\alpha) - n \cdot \alpha)} = I_1 (\alpha) \hspace{0.05cm}.$$
  • Dagegen ist der zweite Integrand eine ungerade Funktion:
$$I_2 (-\alpha) = {\sin( \eta \cdot \sin(-\alpha) + n \cdot \alpha)} = {\sin( -\eta \cdot \sin(\alpha) + n \cdot \alpha)}= -{\sin( \eta \cdot \sin(\alpha) - n \cdot \alpha)} = -I_2 (\alpha) \hspace{0.05cm}.$$
  • Somit verschwindet das zweite Integral und man erhält unter Berücksichtigung der Symmetrie:
$$D_n = \frac{1}{\pi}\cdot \int_{0}^{\pi} {\cos( \eta \cdot \sin(\alpha) - n \cdot \alpha)}\hspace{0.1cm}{\rm d}\alpha \hspace{0.05cm}.$$
  • Richtig ist somit der Lösungsvorschlag 1.


(4)  Entsprechend der iterativen Berechnungsformel gilt für  $η = 2$:

$$ D_2 = D_1 - D_0 = 0.577 - 0.224 \hspace{0.15cm}\underline {= 0.353} \hspace{0.05cm},$$
$$D_3 = 2 \cdot D_2 - D_1 = 2 \cdot 0.353 - 0.577 \hspace{0.15cm}\underline {= 0.129} \hspace{0.05cm}.$$


(5)  Aufgrund der angegebenen Symmetriebeziehung gilt weiter:

$$ D_{–2} = D_2\hspace{0.15cm}\underline {= 0.353} \hspace{0.05cm},$$
$$D_{–3} = -D_3 \hspace{0.15cm}\underline {= -0.129} \hspace{0.05cm}.$$